Find one domain of attraction for this system












1












$begingroup$


Assume the system:
begin{align}
begin{pmatrix}
x \
y \
end{pmatrix}' &= begin{pmatrix}
-(1-y)x \
-(1-x)y \
end{pmatrix}
end{align}

and the Lyapunov function
$$
V(x,y)=x^2+y^2$$



To begin with, the origin is a point of equilibrium and:



$bullet,$$V$ is positive definite since $$V(x,y) >0, quad forall x,y in mathbb{R^2}setminus(0,0)$$ and $V(0,0)=0$.



$bullet,$
$$
dot{V}(x,y)=-2left[x^2-(x^2y+xy^2)+y^2right]=-2left[x^2(1-y)+y^2(1-x)right]$$

For $dot{V}$ to be negative definite, we must have
$$
1-y>0 iff y<1 quadtext{and}quad 1-x<0 iff x<1
$$

Moreover, for the circular disks $x^2+y^2=c$ to lie inside the region:
$$
R=Big{ (x,y)in mathbb{R^2}: x<1,y<1Big}$$

we must have $c<1$. Therefore, one of the system's attraction domains is:
$$
S=Big{(x,y)in mathbb{R^2}: x^2+y^2 < 1Big}
$$



Question: It's very likely that this domain of attraction is not the maximum possible. Are there any ways to obtain the maximum one from the inequality $dot{V}(x,y)<0$? Is it necessary to find the maximum domain or finding a smaller one, which can be derived easily, does the job?










share|cite|improve this question









$endgroup$








  • 1




    $begingroup$
    Whether it's necessary depends on what the "job" is.
    $endgroup$
    – Robert Israel
    Nov 28 '18 at 19:37
















1












$begingroup$


Assume the system:
begin{align}
begin{pmatrix}
x \
y \
end{pmatrix}' &= begin{pmatrix}
-(1-y)x \
-(1-x)y \
end{pmatrix}
end{align}

and the Lyapunov function
$$
V(x,y)=x^2+y^2$$



To begin with, the origin is a point of equilibrium and:



$bullet,$$V$ is positive definite since $$V(x,y) >0, quad forall x,y in mathbb{R^2}setminus(0,0)$$ and $V(0,0)=0$.



$bullet,$
$$
dot{V}(x,y)=-2left[x^2-(x^2y+xy^2)+y^2right]=-2left[x^2(1-y)+y^2(1-x)right]$$

For $dot{V}$ to be negative definite, we must have
$$
1-y>0 iff y<1 quadtext{and}quad 1-x<0 iff x<1
$$

Moreover, for the circular disks $x^2+y^2=c$ to lie inside the region:
$$
R=Big{ (x,y)in mathbb{R^2}: x<1,y<1Big}$$

we must have $c<1$. Therefore, one of the system's attraction domains is:
$$
S=Big{(x,y)in mathbb{R^2}: x^2+y^2 < 1Big}
$$



Question: It's very likely that this domain of attraction is not the maximum possible. Are there any ways to obtain the maximum one from the inequality $dot{V}(x,y)<0$? Is it necessary to find the maximum domain or finding a smaller one, which can be derived easily, does the job?










share|cite|improve this question









$endgroup$








  • 1




    $begingroup$
    Whether it's necessary depends on what the "job" is.
    $endgroup$
    – Robert Israel
    Nov 28 '18 at 19:37














1












1








1





$begingroup$


Assume the system:
begin{align}
begin{pmatrix}
x \
y \
end{pmatrix}' &= begin{pmatrix}
-(1-y)x \
-(1-x)y \
end{pmatrix}
end{align}

and the Lyapunov function
$$
V(x,y)=x^2+y^2$$



To begin with, the origin is a point of equilibrium and:



$bullet,$$V$ is positive definite since $$V(x,y) >0, quad forall x,y in mathbb{R^2}setminus(0,0)$$ and $V(0,0)=0$.



$bullet,$
$$
dot{V}(x,y)=-2left[x^2-(x^2y+xy^2)+y^2right]=-2left[x^2(1-y)+y^2(1-x)right]$$

For $dot{V}$ to be negative definite, we must have
$$
1-y>0 iff y<1 quadtext{and}quad 1-x<0 iff x<1
$$

Moreover, for the circular disks $x^2+y^2=c$ to lie inside the region:
$$
R=Big{ (x,y)in mathbb{R^2}: x<1,y<1Big}$$

we must have $c<1$. Therefore, one of the system's attraction domains is:
$$
S=Big{(x,y)in mathbb{R^2}: x^2+y^2 < 1Big}
$$



Question: It's very likely that this domain of attraction is not the maximum possible. Are there any ways to obtain the maximum one from the inequality $dot{V}(x,y)<0$? Is it necessary to find the maximum domain or finding a smaller one, which can be derived easily, does the job?










share|cite|improve this question









$endgroup$




Assume the system:
begin{align}
begin{pmatrix}
x \
y \
end{pmatrix}' &= begin{pmatrix}
-(1-y)x \
-(1-x)y \
end{pmatrix}
end{align}

and the Lyapunov function
$$
V(x,y)=x^2+y^2$$



To begin with, the origin is a point of equilibrium and:



$bullet,$$V$ is positive definite since $$V(x,y) >0, quad forall x,y in mathbb{R^2}setminus(0,0)$$ and $V(0,0)=0$.



$bullet,$
$$
dot{V}(x,y)=-2left[x^2-(x^2y+xy^2)+y^2right]=-2left[x^2(1-y)+y^2(1-x)right]$$

For $dot{V}$ to be negative definite, we must have
$$
1-y>0 iff y<1 quadtext{and}quad 1-x<0 iff x<1
$$

Moreover, for the circular disks $x^2+y^2=c$ to lie inside the region:
$$
R=Big{ (x,y)in mathbb{R^2}: x<1,y<1Big}$$

we must have $c<1$. Therefore, one of the system's attraction domains is:
$$
S=Big{(x,y)in mathbb{R^2}: x^2+y^2 < 1Big}
$$



Question: It's very likely that this domain of attraction is not the maximum possible. Are there any ways to obtain the maximum one from the inequality $dot{V}(x,y)<0$? Is it necessary to find the maximum domain or finding a smaller one, which can be derived easily, does the job?







differential-equations dynamical-systems stability-in-odes lyapunov-functions






share|cite|improve this question













share|cite|improve this question











share|cite|improve this question




share|cite|improve this question










asked Nov 28 '18 at 19:05









JevautJevaut

1,015112




1,015112








  • 1




    $begingroup$
    Whether it's necessary depends on what the "job" is.
    $endgroup$
    – Robert Israel
    Nov 28 '18 at 19:37














  • 1




    $begingroup$
    Whether it's necessary depends on what the "job" is.
    $endgroup$
    – Robert Israel
    Nov 28 '18 at 19:37








1




1




$begingroup$
Whether it's necessary depends on what the "job" is.
$endgroup$
– Robert Israel
Nov 28 '18 at 19:37




$begingroup$
Whether it's necessary depends on what the "job" is.
$endgroup$
– Robert Israel
Nov 28 '18 at 19:37










2 Answers
2






active

oldest

votes


















1












$begingroup$

In most cases, exercises like this are properly created so you can find the true corresponding attraction domain by using the Lyapunov Function given, since if the Lyapunov Function does the trick then a strictly precise inequality (or conclusion) about its sign leads to the correct assumption. Note that in any general, complicated model, finding a Lyapunov Function is in the first place a very hard and tricky task, let alone finding the exact one that can give you the maximum domain of attraction.



In your exercise though, which is precisely created, the domain of attraction that you have yielded is indeed correct, as it can be seen by the streamplot below, since for $x>1$ and $y>1$ the phase portrait arrows "drag" away :



$qquad qquad qquad$enter image description here






share|cite|improve this answer









$endgroup$













  • $begingroup$
    Are you an ex-student of Staurakakis?!
    $endgroup$
    – Jevaut
    Nov 28 '18 at 19:27










  • $begingroup$
    Yes, that would be true. Why are you asking ?
    $endgroup$
    – Rebellos
    Nov 28 '18 at 19:55










  • $begingroup$
    Sorry for the excess excitement lol. I'm asking cause I'm also one of his students now. I checked your profile and it got me like: "what a small world".
    $endgroup$
    – Jevaut
    Nov 28 '18 at 20:07










  • $begingroup$
    @Jevaut Haha, that's true, indeed a coincidence. How come you remove the accept mark by the way ? Oh, a note for such problems : In this case, as Robert Israel perfectly elaborated below, since you can solve the system in closed form, you can make a safe conclusion. In cases where the system cannot be solved in closed form (which is the point of the dynamical systems course) then what happens is the elaboration of my answer.
    $endgroup$
    – Rebellos
    Nov 28 '18 at 20:11










  • $begingroup$
    @Jevaut Where are you coming from?
    $endgroup$
    – Magic K. Mamba
    Nov 28 '18 at 20:14



















2












$begingroup$

This particular system has closed form solutions. The integral curves are
$x-ln(|x|)-y+ln(|y|) =$constant. In particular, the separatrices forming the
boundary of the "maximal" basin of attraction of $(0,0)$ are the stable manifold of the saddle point $(1,1)$, namely the branches of $x - ln(x) - y + ln(y) = 0$ that are not on $x=y$.



enter image description here






share|cite|improve this answer











$endgroup$













    Your Answer





    StackExchange.ifUsing("editor", function () {
    return StackExchange.using("mathjaxEditing", function () {
    StackExchange.MarkdownEditor.creationCallbacks.add(function (editor, postfix) {
    StackExchange.mathjaxEditing.prepareWmdForMathJax(editor, postfix, [["$", "$"], ["\\(","\\)"]]);
    });
    });
    }, "mathjax-editing");

    StackExchange.ready(function() {
    var channelOptions = {
    tags: "".split(" "),
    id: "69"
    };
    initTagRenderer("".split(" "), "".split(" "), channelOptions);

    StackExchange.using("externalEditor", function() {
    // Have to fire editor after snippets, if snippets enabled
    if (StackExchange.settings.snippets.snippetsEnabled) {
    StackExchange.using("snippets", function() {
    createEditor();
    });
    }
    else {
    createEditor();
    }
    });

    function createEditor() {
    StackExchange.prepareEditor({
    heartbeatType: 'answer',
    autoActivateHeartbeat: false,
    convertImagesToLinks: true,
    noModals: true,
    showLowRepImageUploadWarning: true,
    reputationToPostImages: 10,
    bindNavPrevention: true,
    postfix: "",
    imageUploader: {
    brandingHtml: "Powered by u003ca class="icon-imgur-white" href="https://imgur.com/"u003eu003c/au003e",
    contentPolicyHtml: "User contributions licensed under u003ca href="https://creativecommons.org/licenses/by-sa/3.0/"u003ecc by-sa 3.0 with attribution requiredu003c/au003e u003ca href="https://stackoverflow.com/legal/content-policy"u003e(content policy)u003c/au003e",
    allowUrls: true
    },
    noCode: true, onDemand: true,
    discardSelector: ".discard-answer"
    ,immediatelyShowMarkdownHelp:true
    });


    }
    });














    draft saved

    draft discarded


















    StackExchange.ready(
    function () {
    StackExchange.openid.initPostLogin('.new-post-login', 'https%3a%2f%2fmath.stackexchange.com%2fquestions%2f3017556%2ffind-one-domain-of-attraction-for-this-system%23new-answer', 'question_page');
    }
    );

    Post as a guest















    Required, but never shown

























    2 Answers
    2






    active

    oldest

    votes








    2 Answers
    2






    active

    oldest

    votes









    active

    oldest

    votes






    active

    oldest

    votes









    1












    $begingroup$

    In most cases, exercises like this are properly created so you can find the true corresponding attraction domain by using the Lyapunov Function given, since if the Lyapunov Function does the trick then a strictly precise inequality (or conclusion) about its sign leads to the correct assumption. Note that in any general, complicated model, finding a Lyapunov Function is in the first place a very hard and tricky task, let alone finding the exact one that can give you the maximum domain of attraction.



    In your exercise though, which is precisely created, the domain of attraction that you have yielded is indeed correct, as it can be seen by the streamplot below, since for $x>1$ and $y>1$ the phase portrait arrows "drag" away :



    $qquad qquad qquad$enter image description here






    share|cite|improve this answer









    $endgroup$













    • $begingroup$
      Are you an ex-student of Staurakakis?!
      $endgroup$
      – Jevaut
      Nov 28 '18 at 19:27










    • $begingroup$
      Yes, that would be true. Why are you asking ?
      $endgroup$
      – Rebellos
      Nov 28 '18 at 19:55










    • $begingroup$
      Sorry for the excess excitement lol. I'm asking cause I'm also one of his students now. I checked your profile and it got me like: "what a small world".
      $endgroup$
      – Jevaut
      Nov 28 '18 at 20:07










    • $begingroup$
      @Jevaut Haha, that's true, indeed a coincidence. How come you remove the accept mark by the way ? Oh, a note for such problems : In this case, as Robert Israel perfectly elaborated below, since you can solve the system in closed form, you can make a safe conclusion. In cases where the system cannot be solved in closed form (which is the point of the dynamical systems course) then what happens is the elaboration of my answer.
      $endgroup$
      – Rebellos
      Nov 28 '18 at 20:11










    • $begingroup$
      @Jevaut Where are you coming from?
      $endgroup$
      – Magic K. Mamba
      Nov 28 '18 at 20:14
















    1












    $begingroup$

    In most cases, exercises like this are properly created so you can find the true corresponding attraction domain by using the Lyapunov Function given, since if the Lyapunov Function does the trick then a strictly precise inequality (or conclusion) about its sign leads to the correct assumption. Note that in any general, complicated model, finding a Lyapunov Function is in the first place a very hard and tricky task, let alone finding the exact one that can give you the maximum domain of attraction.



    In your exercise though, which is precisely created, the domain of attraction that you have yielded is indeed correct, as it can be seen by the streamplot below, since for $x>1$ and $y>1$ the phase portrait arrows "drag" away :



    $qquad qquad qquad$enter image description here






    share|cite|improve this answer









    $endgroup$













    • $begingroup$
      Are you an ex-student of Staurakakis?!
      $endgroup$
      – Jevaut
      Nov 28 '18 at 19:27










    • $begingroup$
      Yes, that would be true. Why are you asking ?
      $endgroup$
      – Rebellos
      Nov 28 '18 at 19:55










    • $begingroup$
      Sorry for the excess excitement lol. I'm asking cause I'm also one of his students now. I checked your profile and it got me like: "what a small world".
      $endgroup$
      – Jevaut
      Nov 28 '18 at 20:07










    • $begingroup$
      @Jevaut Haha, that's true, indeed a coincidence. How come you remove the accept mark by the way ? Oh, a note for such problems : In this case, as Robert Israel perfectly elaborated below, since you can solve the system in closed form, you can make a safe conclusion. In cases where the system cannot be solved in closed form (which is the point of the dynamical systems course) then what happens is the elaboration of my answer.
      $endgroup$
      – Rebellos
      Nov 28 '18 at 20:11










    • $begingroup$
      @Jevaut Where are you coming from?
      $endgroup$
      – Magic K. Mamba
      Nov 28 '18 at 20:14














    1












    1








    1





    $begingroup$

    In most cases, exercises like this are properly created so you can find the true corresponding attraction domain by using the Lyapunov Function given, since if the Lyapunov Function does the trick then a strictly precise inequality (or conclusion) about its sign leads to the correct assumption. Note that in any general, complicated model, finding a Lyapunov Function is in the first place a very hard and tricky task, let alone finding the exact one that can give you the maximum domain of attraction.



    In your exercise though, which is precisely created, the domain of attraction that you have yielded is indeed correct, as it can be seen by the streamplot below, since for $x>1$ and $y>1$ the phase portrait arrows "drag" away :



    $qquad qquad qquad$enter image description here






    share|cite|improve this answer









    $endgroup$



    In most cases, exercises like this are properly created so you can find the true corresponding attraction domain by using the Lyapunov Function given, since if the Lyapunov Function does the trick then a strictly precise inequality (or conclusion) about its sign leads to the correct assumption. Note that in any general, complicated model, finding a Lyapunov Function is in the first place a very hard and tricky task, let alone finding the exact one that can give you the maximum domain of attraction.



    In your exercise though, which is precisely created, the domain of attraction that you have yielded is indeed correct, as it can be seen by the streamplot below, since for $x>1$ and $y>1$ the phase portrait arrows "drag" away :



    $qquad qquad qquad$enter image description here







    share|cite|improve this answer












    share|cite|improve this answer



    share|cite|improve this answer










    answered Nov 28 '18 at 19:19









    RebellosRebellos

    14.5k31246




    14.5k31246












    • $begingroup$
      Are you an ex-student of Staurakakis?!
      $endgroup$
      – Jevaut
      Nov 28 '18 at 19:27










    • $begingroup$
      Yes, that would be true. Why are you asking ?
      $endgroup$
      – Rebellos
      Nov 28 '18 at 19:55










    • $begingroup$
      Sorry for the excess excitement lol. I'm asking cause I'm also one of his students now. I checked your profile and it got me like: "what a small world".
      $endgroup$
      – Jevaut
      Nov 28 '18 at 20:07










    • $begingroup$
      @Jevaut Haha, that's true, indeed a coincidence. How come you remove the accept mark by the way ? Oh, a note for such problems : In this case, as Robert Israel perfectly elaborated below, since you can solve the system in closed form, you can make a safe conclusion. In cases where the system cannot be solved in closed form (which is the point of the dynamical systems course) then what happens is the elaboration of my answer.
      $endgroup$
      – Rebellos
      Nov 28 '18 at 20:11










    • $begingroup$
      @Jevaut Where are you coming from?
      $endgroup$
      – Magic K. Mamba
      Nov 28 '18 at 20:14


















    • $begingroup$
      Are you an ex-student of Staurakakis?!
      $endgroup$
      – Jevaut
      Nov 28 '18 at 19:27










    • $begingroup$
      Yes, that would be true. Why are you asking ?
      $endgroup$
      – Rebellos
      Nov 28 '18 at 19:55










    • $begingroup$
      Sorry for the excess excitement lol. I'm asking cause I'm also one of his students now. I checked your profile and it got me like: "what a small world".
      $endgroup$
      – Jevaut
      Nov 28 '18 at 20:07










    • $begingroup$
      @Jevaut Haha, that's true, indeed a coincidence. How come you remove the accept mark by the way ? Oh, a note for such problems : In this case, as Robert Israel perfectly elaborated below, since you can solve the system in closed form, you can make a safe conclusion. In cases where the system cannot be solved in closed form (which is the point of the dynamical systems course) then what happens is the elaboration of my answer.
      $endgroup$
      – Rebellos
      Nov 28 '18 at 20:11










    • $begingroup$
      @Jevaut Where are you coming from?
      $endgroup$
      – Magic K. Mamba
      Nov 28 '18 at 20:14
















    $begingroup$
    Are you an ex-student of Staurakakis?!
    $endgroup$
    – Jevaut
    Nov 28 '18 at 19:27




    $begingroup$
    Are you an ex-student of Staurakakis?!
    $endgroup$
    – Jevaut
    Nov 28 '18 at 19:27












    $begingroup$
    Yes, that would be true. Why are you asking ?
    $endgroup$
    – Rebellos
    Nov 28 '18 at 19:55




    $begingroup$
    Yes, that would be true. Why are you asking ?
    $endgroup$
    – Rebellos
    Nov 28 '18 at 19:55












    $begingroup$
    Sorry for the excess excitement lol. I'm asking cause I'm also one of his students now. I checked your profile and it got me like: "what a small world".
    $endgroup$
    – Jevaut
    Nov 28 '18 at 20:07




    $begingroup$
    Sorry for the excess excitement lol. I'm asking cause I'm also one of his students now. I checked your profile and it got me like: "what a small world".
    $endgroup$
    – Jevaut
    Nov 28 '18 at 20:07












    $begingroup$
    @Jevaut Haha, that's true, indeed a coincidence. How come you remove the accept mark by the way ? Oh, a note for such problems : In this case, as Robert Israel perfectly elaborated below, since you can solve the system in closed form, you can make a safe conclusion. In cases where the system cannot be solved in closed form (which is the point of the dynamical systems course) then what happens is the elaboration of my answer.
    $endgroup$
    – Rebellos
    Nov 28 '18 at 20:11




    $begingroup$
    @Jevaut Haha, that's true, indeed a coincidence. How come you remove the accept mark by the way ? Oh, a note for such problems : In this case, as Robert Israel perfectly elaborated below, since you can solve the system in closed form, you can make a safe conclusion. In cases where the system cannot be solved in closed form (which is the point of the dynamical systems course) then what happens is the elaboration of my answer.
    $endgroup$
    – Rebellos
    Nov 28 '18 at 20:11












    $begingroup$
    @Jevaut Where are you coming from?
    $endgroup$
    – Magic K. Mamba
    Nov 28 '18 at 20:14




    $begingroup$
    @Jevaut Where are you coming from?
    $endgroup$
    – Magic K. Mamba
    Nov 28 '18 at 20:14











    2












    $begingroup$

    This particular system has closed form solutions. The integral curves are
    $x-ln(|x|)-y+ln(|y|) =$constant. In particular, the separatrices forming the
    boundary of the "maximal" basin of attraction of $(0,0)$ are the stable manifold of the saddle point $(1,1)$, namely the branches of $x - ln(x) - y + ln(y) = 0$ that are not on $x=y$.



    enter image description here






    share|cite|improve this answer











    $endgroup$


















      2












      $begingroup$

      This particular system has closed form solutions. The integral curves are
      $x-ln(|x|)-y+ln(|y|) =$constant. In particular, the separatrices forming the
      boundary of the "maximal" basin of attraction of $(0,0)$ are the stable manifold of the saddle point $(1,1)$, namely the branches of $x - ln(x) - y + ln(y) = 0$ that are not on $x=y$.



      enter image description here






      share|cite|improve this answer











      $endgroup$
















        2












        2








        2





        $begingroup$

        This particular system has closed form solutions. The integral curves are
        $x-ln(|x|)-y+ln(|y|) =$constant. In particular, the separatrices forming the
        boundary of the "maximal" basin of attraction of $(0,0)$ are the stable manifold of the saddle point $(1,1)$, namely the branches of $x - ln(x) - y + ln(y) = 0$ that are not on $x=y$.



        enter image description here






        share|cite|improve this answer











        $endgroup$



        This particular system has closed form solutions. The integral curves are
        $x-ln(|x|)-y+ln(|y|) =$constant. In particular, the separatrices forming the
        boundary of the "maximal" basin of attraction of $(0,0)$ are the stable manifold of the saddle point $(1,1)$, namely the branches of $x - ln(x) - y + ln(y) = 0$ that are not on $x=y$.



        enter image description here







        share|cite|improve this answer














        share|cite|improve this answer



        share|cite|improve this answer








        edited Nov 28 '18 at 19:35

























        answered Nov 28 '18 at 19:28









        Robert IsraelRobert Israel

        319k23209459




        319k23209459






























            draft saved

            draft discarded




















































            Thanks for contributing an answer to Mathematics Stack Exchange!


            • Please be sure to answer the question. Provide details and share your research!

            But avoid



            • Asking for help, clarification, or responding to other answers.

            • Making statements based on opinion; back them up with references or personal experience.


            Use MathJax to format equations. MathJax reference.


            To learn more, see our tips on writing great answers.




            draft saved


            draft discarded














            StackExchange.ready(
            function () {
            StackExchange.openid.initPostLogin('.new-post-login', 'https%3a%2f%2fmath.stackexchange.com%2fquestions%2f3017556%2ffind-one-domain-of-attraction-for-this-system%23new-answer', 'question_page');
            }
            );

            Post as a guest















            Required, but never shown





















































            Required, but never shown














            Required, but never shown












            Required, but never shown







            Required, but never shown

































            Required, but never shown














            Required, but never shown












            Required, but never shown







            Required, but never shown







            Popular posts from this blog

            Plaza Victoria

            In PowerPoint, is there a keyboard shortcut for bulleted / numbered list?

            How to put 3 figures in Latex with 2 figures side by side and 1 below these side by side images but in...